Menu Close

Question-203115




Question Number 203115 by Mathstar last updated on 10/Jan/24
Commented by Mathstar last updated on 10/Jan/24
Send an explanatory solution, thank you
$$\mathrm{Send}\:\mathrm{an}\:\mathrm{explanatory}\:\mathrm{solution},\:\mathrm{thank}\:\mathrm{you} \\ $$
Answered by mr W last updated on 10/Jan/24
y=x^2 e^x   y′=e^x (x^2 +2x)  center of circle (0,r)  say the circle tangents the curve  additionally at P(p,q).  q=p^2 e^p   tan θ=y′=e^p (p^2 +2p)  p=r sin θ=2r sin (θ/2) cos (θ/2)  q=r(1−cos θ)=2r sin^2  (θ/2)  tan (θ/2)=(q/p)=((p^2 e^p )/p)=pe^p   tan θ=((2 tan (θ/2))/(1−tan^2  (θ/2)))  e^p (p^2 +2p)=((2pe^p )/(1−p^2 e^(2p) ))  ⇒(p+2)(1−p^2 e^(2p) )=2  ⇒p=0 ∨ p≈0.2619  r_(max) =(p/(sin θ))=(√(p^2 +(p^2 /(tan^2  θ))))    =(√(p^2 +(1/(e^(2p) (p+2)^2 ))))≈0.4294
$${y}={x}^{\mathrm{2}} {e}^{{x}} \\ $$$${y}'={e}^{{x}} \left({x}^{\mathrm{2}} +\mathrm{2}{x}\right) \\ $$$${center}\:{of}\:{circle}\:\left(\mathrm{0},{r}\right) \\ $$$${say}\:{the}\:{circle}\:{tangents}\:{the}\:{curve} \\ $$$${additionally}\:{at}\:{P}\left({p},{q}\right). \\ $$$${q}={p}^{\mathrm{2}} {e}^{{p}} \\ $$$$\mathrm{tan}\:\theta={y}'={e}^{{p}} \left({p}^{\mathrm{2}} +\mathrm{2}{p}\right) \\ $$$${p}={r}\:\mathrm{sin}\:\theta=\mathrm{2}{r}\:\mathrm{sin}\:\frac{\theta}{\mathrm{2}}\:\mathrm{cos}\:\frac{\theta}{\mathrm{2}} \\ $$$${q}={r}\left(\mathrm{1}−\mathrm{cos}\:\theta\right)=\mathrm{2}{r}\:\mathrm{sin}^{\mathrm{2}} \:\frac{\theta}{\mathrm{2}} \\ $$$$\mathrm{tan}\:\frac{\theta}{\mathrm{2}}=\frac{{q}}{{p}}=\frac{{p}^{\mathrm{2}} {e}^{{p}} }{{p}}={pe}^{{p}} \\ $$$$\mathrm{tan}\:\theta=\frac{\mathrm{2}\:\mathrm{tan}\:\frac{\theta}{\mathrm{2}}}{\mathrm{1}−\mathrm{tan}^{\mathrm{2}} \:\frac{\theta}{\mathrm{2}}} \\ $$$${e}^{{p}} \left({p}^{\mathrm{2}} +\mathrm{2}{p}\right)=\frac{\mathrm{2}{pe}^{{p}} }{\mathrm{1}−{p}^{\mathrm{2}} {e}^{\mathrm{2}{p}} } \\ $$$$\Rightarrow\left({p}+\mathrm{2}\right)\left(\mathrm{1}−{p}^{\mathrm{2}} {e}^{\mathrm{2}{p}} \right)=\mathrm{2} \\ $$$$\Rightarrow{p}=\mathrm{0}\:\vee\:{p}\approx\mathrm{0}.\mathrm{2619} \\ $$$${r}_{{max}} =\frac{{p}}{\mathrm{sin}\:\theta}=\sqrt{{p}^{\mathrm{2}} +\frac{{p}^{\mathrm{2}} }{\mathrm{tan}^{\mathrm{2}} \:\theta}} \\ $$$$\:\:=\sqrt{{p}^{\mathrm{2}} +\frac{\mathrm{1}}{{e}^{\mathrm{2}{p}} \left({p}+\mathrm{2}\right)^{\mathrm{2}} }}\approx\mathrm{0}.\mathrm{4294} \\ $$
Commented by Mathstar last updated on 10/Jan/24
Thank you. I will look out for a video that can explain better.
Commented by mr W last updated on 10/Jan/24
Commented by mr W last updated on 10/Jan/24
Algebra  Geometry  Trigonometry  Derivative  Equations
$${Algebra} \\ $$$${Geometry} \\ $$$${Trigonometry} \\ $$$${Derivative} \\ $$$${Equations} \\ $$
Commented by Mathstar last updated on 10/Jan/24
Thank you so much Mr W. Please how can I understand something like this?
Commented by Mathstar last updated on 10/Jan/24
Could you please enumerate the scope in this type of question?
Commented by mr W last updated on 11/Jan/24
Commented by mr W last updated on 11/Jan/24
such that a circle only touches a  curve at a point, the radius of the  circle may not be larger than the  radius R of the osculating circle  with R=∣(((1+y′^2 )^(3/2) )/(y′′))∣.  in our case R=0.5.  but this circle may intersect the  curve at other points. such that  this doesn′t happen, we must   ensure that the circle which touches  the curce at point (0,0) maximally  touches the curve at an other point.  this circle is then the largest circle  which fulfills the requirement of  the question.  this is the “theory”behind my  solution.
$${such}\:{that}\:{a}\:{circle}\:{only}\:{touches}\:{a} \\ $$$${curve}\:{at}\:{a}\:{point},\:{the}\:{radius}\:{of}\:{the} \\ $$$${circle}\:{may}\:{not}\:{be}\:{larger}\:{than}\:{the} \\ $$$${radius}\:{R}\:{of}\:{the}\:{osculating}\:{circle} \\ $$$${with}\:{R}=\mid\frac{\left(\mathrm{1}+{y}'^{\mathrm{2}} \right)^{\frac{\mathrm{3}}{\mathrm{2}}} }{{y}''}\mid. \\ $$$${in}\:{our}\:{case}\:{R}=\mathrm{0}.\mathrm{5}. \\ $$$${but}\:{this}\:{circle}\:{may}\:{intersect}\:{the} \\ $$$${curve}\:{at}\:{other}\:{points}.\:{such}\:{that} \\ $$$${this}\:{doesn}'{t}\:{happen},\:{we}\:{must}\: \\ $$$${ensure}\:{that}\:{the}\:{circle}\:{which}\:{touches} \\ $$$${the}\:{curce}\:{at}\:{point}\:\left(\mathrm{0},\mathrm{0}\right)\:{maximally} \\ $$$${touches}\:{the}\:{curve}\:{at}\:{an}\:{other}\:{point}. \\ $$$${this}\:{circle}\:{is}\:{then}\:{the}\:{largest}\:{circle} \\ $$$${which}\:{fulfills}\:{the}\:{requirement}\:{of} \\ $$$${the}\:{question}. \\ $$$${this}\:{is}\:{the}\:“{theory}''{behind}\:{my} \\ $$$${solution}. \\ $$
Commented by mr W last updated on 11/Jan/24
Commented by Mathstar last updated on 11/Jan/24
Thank you so much, Mr W. You've been of great support to me ever since I joined this platform ��‍♂️

Leave a Reply

Your email address will not be published. Required fields are marked *